Proving polynomial over Q is irreducible












1












$begingroup$



Prove that if $a$ and $b$ are odd then the polynomial $$x^3+ax+b$$is irreducible over $mathbb{Q}$




I would be very much thankful if someone could help me with this one.










share|cite|improve this question











$endgroup$












  • $begingroup$
    Eisenstein criterion. Its not ''linear algebra''.
    $endgroup$
    – Wuestenfux
    Dec 9 '18 at 9:42








  • 4




    $begingroup$
    reduction modulo 2 is the obvious approach. Just show that $x^3+x+1$ is irreducible over $mathbb Z_2$
    $endgroup$
    – Peter
    Dec 9 '18 at 9:53


















1












$begingroup$



Prove that if $a$ and $b$ are odd then the polynomial $$x^3+ax+b$$is irreducible over $mathbb{Q}$




I would be very much thankful if someone could help me with this one.










share|cite|improve this question











$endgroup$












  • $begingroup$
    Eisenstein criterion. Its not ''linear algebra''.
    $endgroup$
    – Wuestenfux
    Dec 9 '18 at 9:42








  • 4




    $begingroup$
    reduction modulo 2 is the obvious approach. Just show that $x^3+x+1$ is irreducible over $mathbb Z_2$
    $endgroup$
    – Peter
    Dec 9 '18 at 9:53
















1












1








1


0



$begingroup$



Prove that if $a$ and $b$ are odd then the polynomial $$x^3+ax+b$$is irreducible over $mathbb{Q}$




I would be very much thankful if someone could help me with this one.










share|cite|improve this question











$endgroup$





Prove that if $a$ and $b$ are odd then the polynomial $$x^3+ax+b$$is irreducible over $mathbb{Q}$




I would be very much thankful if someone could help me with this one.







abstract-algebra elementary-number-theory polynomials






share|cite|improve this question















share|cite|improve this question













share|cite|improve this question




share|cite|improve this question








edited Dec 9 '18 at 10:11









greedoid

42.7k1153105




42.7k1153105










asked Dec 9 '18 at 9:38







user615503



















  • $begingroup$
    Eisenstein criterion. Its not ''linear algebra''.
    $endgroup$
    – Wuestenfux
    Dec 9 '18 at 9:42








  • 4




    $begingroup$
    reduction modulo 2 is the obvious approach. Just show that $x^3+x+1$ is irreducible over $mathbb Z_2$
    $endgroup$
    – Peter
    Dec 9 '18 at 9:53




















  • $begingroup$
    Eisenstein criterion. Its not ''linear algebra''.
    $endgroup$
    – Wuestenfux
    Dec 9 '18 at 9:42








  • 4




    $begingroup$
    reduction modulo 2 is the obvious approach. Just show that $x^3+x+1$ is irreducible over $mathbb Z_2$
    $endgroup$
    – Peter
    Dec 9 '18 at 9:53


















$begingroup$
Eisenstein criterion. Its not ''linear algebra''.
$endgroup$
– Wuestenfux
Dec 9 '18 at 9:42






$begingroup$
Eisenstein criterion. Its not ''linear algebra''.
$endgroup$
– Wuestenfux
Dec 9 '18 at 9:42






4




4




$begingroup$
reduction modulo 2 is the obvious approach. Just show that $x^3+x+1$ is irreducible over $mathbb Z_2$
$endgroup$
– Peter
Dec 9 '18 at 9:53






$begingroup$
reduction modulo 2 is the obvious approach. Just show that $x^3+x+1$ is irreducible over $mathbb Z_2$
$endgroup$
– Peter
Dec 9 '18 at 9:53












2 Answers
2






active

oldest

votes


















2












$begingroup$

Say it is reducibile, then it has to have rational root and it has to be even an integer because leading coeficient is $1$. So we have $$(x^2+cx+d)(x+r) = x^3+ax+b$$



where $c,d,rin mathbb{Z}$. Since $rd = b$ and thus $r,d$ are both odd.



Now we have also $c+r =0$ (so $c$ is odd) and $cr+d =a$. A contradiction.






share|cite|improve this answer









$endgroup$





















    3












    $begingroup$

    A more direct approach might be to show that it is irreducible modulo $p$. Once we've shown that, it is surely irreducible over $mathbb Z$ and hence $mathbb Q$ as well (because the leading coefficient is $1$; by the rational root theorem, any rational root is integral). Since we are given $a,b$ are odd, the obvious choice is to take modulo $2$, where we have, in $mathbb Z_2$,
    $$x^3+ax+b=x^3+x+1.$$
    Suppose this is reducible, then it has a root over $mathbb Z_2$, and this must be either $0$ or $1$. But $0^3+0+1neq0$ and $1^3+1+1=1neq0$, so neither of the only two elements in $mathbb Z_2$ are a root. We conclude that it is irreducible over $mathbb Z_2$, hence over $mathbb Q$ as well.






    share|cite|improve this answer









    $endgroup$













      Your Answer





      StackExchange.ifUsing("editor", function () {
      return StackExchange.using("mathjaxEditing", function () {
      StackExchange.MarkdownEditor.creationCallbacks.add(function (editor, postfix) {
      StackExchange.mathjaxEditing.prepareWmdForMathJax(editor, postfix, [["$", "$"], ["\\(","\\)"]]);
      });
      });
      }, "mathjax-editing");

      StackExchange.ready(function() {
      var channelOptions = {
      tags: "".split(" "),
      id: "69"
      };
      initTagRenderer("".split(" "), "".split(" "), channelOptions);

      StackExchange.using("externalEditor", function() {
      // Have to fire editor after snippets, if snippets enabled
      if (StackExchange.settings.snippets.snippetsEnabled) {
      StackExchange.using("snippets", function() {
      createEditor();
      });
      }
      else {
      createEditor();
      }
      });

      function createEditor() {
      StackExchange.prepareEditor({
      heartbeatType: 'answer',
      autoActivateHeartbeat: false,
      convertImagesToLinks: true,
      noModals: true,
      showLowRepImageUploadWarning: true,
      reputationToPostImages: 10,
      bindNavPrevention: true,
      postfix: "",
      imageUploader: {
      brandingHtml: "Powered by u003ca class="icon-imgur-white" href="https://imgur.com/"u003eu003c/au003e",
      contentPolicyHtml: "User contributions licensed under u003ca href="https://creativecommons.org/licenses/by-sa/3.0/"u003ecc by-sa 3.0 with attribution requiredu003c/au003e u003ca href="https://stackoverflow.com/legal/content-policy"u003e(content policy)u003c/au003e",
      allowUrls: true
      },
      noCode: true, onDemand: true,
      discardSelector: ".discard-answer"
      ,immediatelyShowMarkdownHelp:true
      });


      }
      });














      draft saved

      draft discarded


















      StackExchange.ready(
      function () {
      StackExchange.openid.initPostLogin('.new-post-login', 'https%3a%2f%2fmath.stackexchange.com%2fquestions%2f3032203%2fproving-polynomial-over-q-is-irreducible%23new-answer', 'question_page');
      }
      );

      Post as a guest















      Required, but never shown
























      2 Answers
      2






      active

      oldest

      votes








      2 Answers
      2






      active

      oldest

      votes









      active

      oldest

      votes






      active

      oldest

      votes









      2












      $begingroup$

      Say it is reducibile, then it has to have rational root and it has to be even an integer because leading coeficient is $1$. So we have $$(x^2+cx+d)(x+r) = x^3+ax+b$$



      where $c,d,rin mathbb{Z}$. Since $rd = b$ and thus $r,d$ are both odd.



      Now we have also $c+r =0$ (so $c$ is odd) and $cr+d =a$. A contradiction.






      share|cite|improve this answer









      $endgroup$


















        2












        $begingroup$

        Say it is reducibile, then it has to have rational root and it has to be even an integer because leading coeficient is $1$. So we have $$(x^2+cx+d)(x+r) = x^3+ax+b$$



        where $c,d,rin mathbb{Z}$. Since $rd = b$ and thus $r,d$ are both odd.



        Now we have also $c+r =0$ (so $c$ is odd) and $cr+d =a$. A contradiction.






        share|cite|improve this answer









        $endgroup$
















          2












          2








          2





          $begingroup$

          Say it is reducibile, then it has to have rational root and it has to be even an integer because leading coeficient is $1$. So we have $$(x^2+cx+d)(x+r) = x^3+ax+b$$



          where $c,d,rin mathbb{Z}$. Since $rd = b$ and thus $r,d$ are both odd.



          Now we have also $c+r =0$ (so $c$ is odd) and $cr+d =a$. A contradiction.






          share|cite|improve this answer









          $endgroup$



          Say it is reducibile, then it has to have rational root and it has to be even an integer because leading coeficient is $1$. So we have $$(x^2+cx+d)(x+r) = x^3+ax+b$$



          where $c,d,rin mathbb{Z}$. Since $rd = b$ and thus $r,d$ are both odd.



          Now we have also $c+r =0$ (so $c$ is odd) and $cr+d =a$. A contradiction.







          share|cite|improve this answer












          share|cite|improve this answer



          share|cite|improve this answer










          answered Dec 9 '18 at 10:00









          greedoidgreedoid

          42.7k1153105




          42.7k1153105























              3












              $begingroup$

              A more direct approach might be to show that it is irreducible modulo $p$. Once we've shown that, it is surely irreducible over $mathbb Z$ and hence $mathbb Q$ as well (because the leading coefficient is $1$; by the rational root theorem, any rational root is integral). Since we are given $a,b$ are odd, the obvious choice is to take modulo $2$, where we have, in $mathbb Z_2$,
              $$x^3+ax+b=x^3+x+1.$$
              Suppose this is reducible, then it has a root over $mathbb Z_2$, and this must be either $0$ or $1$. But $0^3+0+1neq0$ and $1^3+1+1=1neq0$, so neither of the only two elements in $mathbb Z_2$ are a root. We conclude that it is irreducible over $mathbb Z_2$, hence over $mathbb Q$ as well.






              share|cite|improve this answer









              $endgroup$


















                3












                $begingroup$

                A more direct approach might be to show that it is irreducible modulo $p$. Once we've shown that, it is surely irreducible over $mathbb Z$ and hence $mathbb Q$ as well (because the leading coefficient is $1$; by the rational root theorem, any rational root is integral). Since we are given $a,b$ are odd, the obvious choice is to take modulo $2$, where we have, in $mathbb Z_2$,
                $$x^3+ax+b=x^3+x+1.$$
                Suppose this is reducible, then it has a root over $mathbb Z_2$, and this must be either $0$ or $1$. But $0^3+0+1neq0$ and $1^3+1+1=1neq0$, so neither of the only two elements in $mathbb Z_2$ are a root. We conclude that it is irreducible over $mathbb Z_2$, hence over $mathbb Q$ as well.






                share|cite|improve this answer









                $endgroup$
















                  3












                  3








                  3





                  $begingroup$

                  A more direct approach might be to show that it is irreducible modulo $p$. Once we've shown that, it is surely irreducible over $mathbb Z$ and hence $mathbb Q$ as well (because the leading coefficient is $1$; by the rational root theorem, any rational root is integral). Since we are given $a,b$ are odd, the obvious choice is to take modulo $2$, where we have, in $mathbb Z_2$,
                  $$x^3+ax+b=x^3+x+1.$$
                  Suppose this is reducible, then it has a root over $mathbb Z_2$, and this must be either $0$ or $1$. But $0^3+0+1neq0$ and $1^3+1+1=1neq0$, so neither of the only two elements in $mathbb Z_2$ are a root. We conclude that it is irreducible over $mathbb Z_2$, hence over $mathbb Q$ as well.






                  share|cite|improve this answer









                  $endgroup$



                  A more direct approach might be to show that it is irreducible modulo $p$. Once we've shown that, it is surely irreducible over $mathbb Z$ and hence $mathbb Q$ as well (because the leading coefficient is $1$; by the rational root theorem, any rational root is integral). Since we are given $a,b$ are odd, the obvious choice is to take modulo $2$, where we have, in $mathbb Z_2$,
                  $$x^3+ax+b=x^3+x+1.$$
                  Suppose this is reducible, then it has a root over $mathbb Z_2$, and this must be either $0$ or $1$. But $0^3+0+1neq0$ and $1^3+1+1=1neq0$, so neither of the only two elements in $mathbb Z_2$ are a root. We conclude that it is irreducible over $mathbb Z_2$, hence over $mathbb Q$ as well.







                  share|cite|improve this answer












                  share|cite|improve this answer



                  share|cite|improve this answer










                  answered Dec 9 '18 at 12:02









                  YiFanYiFan

                  3,9741627




                  3,9741627






























                      draft saved

                      draft discarded




















































                      Thanks for contributing an answer to Mathematics Stack Exchange!


                      • Please be sure to answer the question. Provide details and share your research!

                      But avoid



                      • Asking for help, clarification, or responding to other answers.

                      • Making statements based on opinion; back them up with references or personal experience.


                      Use MathJax to format equations. MathJax reference.


                      To learn more, see our tips on writing great answers.




                      draft saved


                      draft discarded














                      StackExchange.ready(
                      function () {
                      StackExchange.openid.initPostLogin('.new-post-login', 'https%3a%2f%2fmath.stackexchange.com%2fquestions%2f3032203%2fproving-polynomial-over-q-is-irreducible%23new-answer', 'question_page');
                      }
                      );

                      Post as a guest















                      Required, but never shown





















































                      Required, but never shown














                      Required, but never shown












                      Required, but never shown







                      Required, but never shown

































                      Required, but never shown














                      Required, but never shown












                      Required, but never shown







                      Required, but never shown







                      Popular posts from this blog

                      Plaza Victoria

                      In PowerPoint, is there a keyboard shortcut for bulleted / numbered list?

                      How to put 3 figures in Latex with 2 figures side by side and 1 below these side by side images but in...